Diễn Đàn MathScopeDiễn Đàn MathScope
  Diễn Đàn MathScope
Ghi Danh Hỏi/Ðáp Thành Viên Social Groups Lịch Ðánh Dấu Ðã Ðọc

Go Back   Diễn Đàn MathScope > Sơ Cấp > Đại Số và Lượng Giác > Các Bài Toán Đã Được Giải

News & Announcements

Ngoài một số quy định đã được nêu trong phần Quy định của Ghi Danh , mọi người tranh thủ bỏ ra 5 phút để đọc thêm một số Quy định sau để khỏi bị treo nick ở MathScope nhé !

* Nội quy MathScope.Org

* Một số quy định chung !

* Quy định về việc viết bài trong diễn đàn MathScope

* Nếu bạn muốn gia nhập đội ngũ BQT thì vui lòng tham gia tại đây

* Những câu hỏi thường gặp

* Về việc viết bài trong Box Đại học và Sau đại học


Ðề tài đã khoá Gởi Ðề Tài Mới
 
Ðiều Chỉnh Xếp Bài
Old 20-08-2010, 10:21 AM   #1
353535
Banned
 
Tham gia ngày: Jul 2010
Đến từ: LVT_NB
Bài gởi: 134
Thanks: 3
Thanked 61 Times in 38 Posts
Gửi tin nhắn qua Yahoo chát tới 353535
Topic về bất đẳng thức

1) Cho $a \ge 4;b \ge 5 ; c \ge 6 $ và $a^2+b^2+c^2=90 $.tìm MIN của: a+b+c
2) Cho 3 số dương a,b,c có a+b+c=1.Tìm MAX của:
$A=ab+ac+bc+ \frac{5}{2}[(a+b)\sqrt{ab}+(b+c)\sqrt{bc}+(c+a)\sqrt{ca}] $
[RIGHT][I][B]Nguồn: MathScope.ORG[/B][/I][/RIGHT]
 

thay đổi nội dung bởi: novae, 22-08-2010 lúc 11:26 AM
353535 is offline  
The Following 17 Users Say Thank You to 353535 For This Useful Post:
duonglangquyen (03-12-2010), fantatista1995 (10-07-2011), hanamichi1302 (03-11-2011), hoangnamb (24-11-2010), IMO 2010 (27-11-2010), Ino_chan (18-12-2010), je.triste (24-02-2011), Kém Toán (17-06-2011), Lê Quang Đức (11-05-2011), mnnn (11-01-2011), ngoduchung8A (13-07-2012), nguyenxuanthai (07-03-2011), nhat7d (27-05-2011), rewrite (10-03-2011), tangocmai (10-11-2012), thanhluan_LTTM (27-02-2011), Yucio.3bi_love (30-06-2011)
Old 20-08-2010, 08:05 PM   #2
khanh.kid
Banned
 
Tham gia ngày: Oct 2009
Bài gởi: 51
Thanks: 16
Thanked 20 Times in 12 Posts
Một bài bất đẳng thức

Cho x,y,z>0,xyz=1
c/m
$18( \frac{1}{x^3 +1}+\frac{1}{y^3 +1}+\frac{1}{z^3 +1}) \le (x+y+z)^3 $
[RIGHT][I][B]Nguồn: MathScope.ORG[/B][/I][/RIGHT]
 

thay đổi nội dung bởi: khanh.kid, 20-08-2010 lúc 08:12 PM
khanh.kid is offline  
The Following 5 Users Say Thank You to khanh.kid For This Useful Post:
hoangnamb (24-11-2010), IMO 2010 (27-11-2010), Ino_chan (18-12-2010), ngoduchung8A (13-07-2012), nhat7d (15-06-2011)
Old 20-08-2010, 08:08 PM   #3
novae
+Thành Viên Danh Dự+
 
novae's Avatar
 
Tham gia ngày: Jul 2010
Đến từ: Event horizon
Bài gởi: 2,453
Thanks: 53
Thanked 3,057 Times in 1,288 Posts
điều kiện giữa x, y, z là gì?
[RIGHT][I][B]Nguồn: MathScope.ORG[/B][/I][/RIGHT]
 
__________________
M.
novae is offline  
The Following 7 Users Say Thank You to novae For This Useful Post:
hanamichi1302 (03-11-2011), hoangnamb (24-11-2010), IMO 2010 (27-11-2010), Ino_chan (18-12-2010), ngoduchung8A (13-07-2012), tienanh_tx (22-04-2012), Unknowing (17-11-2010)
Old 20-08-2010, 08:12 PM   #4
khanh.kid
Banned
 
Tham gia ngày: Oct 2009
Bài gởi: 51
Thanks: 16
Thanked 20 Times in 12 Posts
à em quên xyz=1
[RIGHT][I][B]Nguồn: MathScope.ORG[/B][/I][/RIGHT]
 
khanh.kid is offline  
The Following 5 Users Say Thank You to khanh.kid For This Useful Post:
hoangnamb (24-11-2010), IMO 2010 (27-11-2010), Ino_chan (18-12-2010), ngoduchung8A (13-07-2012), nguyenhuuthang (15-01-2011)
Old 20-08-2010, 10:02 PM   #5
trungthptpb
+Thành Viên+
 
Tham gia ngày: Aug 2010
Bài gởi: 1
Thanks: 0
Thanked 5 Times in 1 Post
Th Mini Natal Bất đẳng thức

Cho x+y+z=6 và x, y, z>0. CMR $8^x+8^y+8^z \ge 4^{x+1}+4^{y+1}+4^{z+1} $

học gõ Latex tại đây: [Only registered and activated users can see links. ]
[RIGHT][I][B]Nguồn: MathScope.ORG[/B][/I][/RIGHT]
 

thay đổi nội dung bởi: novae, 20-08-2010 lúc 10:13 PM
trungthptpb is offline  
The Following 5 Users Say Thank You to trungthptpb For This Useful Post:
hoangnamb (24-11-2010), IMO 2010 (27-11-2010), Ino_chan (18-12-2010), ngoduchung8A (13-07-2012), vuquyen93 (14-09-2010)
Old 20-08-2010, 10:44 PM   #6
shinomoriaoshi
+Thành Viên+
 
Tham gia ngày: Mar 2010
Đến từ: Tuy Hòa
Bài gởi: 198
Thanks: 198
Thanked 129 Times in 72 Posts
Bài này thì theo mình là đặt $a=2^x; b=2^y; c=2^z $ rồi dùng điểm rơi Côsi
[RIGHT][I][B]Nguồn: MathScope.ORG[/B][/I][/RIGHT]
 
shinomoriaoshi is offline  
The Following 4 Users Say Thank You to shinomoriaoshi For This Useful Post:
hanamichi1302 (03-11-2011), hoangnamb (24-11-2010), IMO 2010 (27-11-2010), ngoduchung8A (13-07-2012)
Old 21-08-2010, 01:43 AM   #7
huynhcongbang
Administrator

 
huynhcongbang's Avatar
 
Tham gia ngày: Feb 2009
Đến từ: Ho Chi Minh City
Bài gởi: 2,413
Thanks: 2,165
Thanked 4,188 Times in 1,381 Posts
Gửi tin nhắn qua Yahoo chát tới huynhcongbang
Trích:
Nguyên văn bởi trungthptpb View Post
Cho x+y+z=6 và x, y, z>0. CMR $8^x+8^y+8^z \ge 4^{x+1}+4^{y+1}+4^{z+1} $
Bài này hình như trước đây có trong cuốn "Bộ đề tuyển sinh" của Bộ GD-ĐT, cũng từng được dùng làm đề thi ở nhiều nơi rồi. Một bài rất quen thuộc!
Như ý giải của bạn shinomoriaoshi ở trên, mình tiếp 1 chút như sau:
Sau khi đặt như thế thì điều kiện đã cho viết lại là:
$a,b,c>0, 2^{x+y+z}=64\Leftrightarrow abc=64 $.
Cần chứng minh rằng:
$a^3+b^3+c^3 \ge 4(a^2+b^2+c^2) $.
Ta có:
$a^3+a^3+64 \ge 3.\sqrt[3]{64a^6}=12a^2\Leftrightarrow a^3+32 \ge 6a^2 $.
Tương tự cho các đánh giá với b, c. Cộng lại theo từng vế, ta được:
$a^3+b^3+c^3+96 \ge 6(a^2+b^2+c^2) $.
Hơn nữa:
$2(a^2+b^2+c^2)\ge 6.\sqrt[3]{a^2b^2c^2}=6.\sqrt[3]{64^2}=96 $.
Tiếp tục cộng hai BĐT này lại, ta có đpcm.
Đẳng thức xảy ra khi và chỉ khi $a=b=c=4 $ hay $x=y=z=2 $.
[RIGHT][I][B]Nguồn: MathScope.ORG[/B][/I][/RIGHT]
 
huynhcongbang is offline  
The Following 13 Users Say Thank You to huynhcongbang For This Useful Post:
ha linh (02-02-2011), hanamichi1302 (03-11-2011), hoangnamb (24-11-2010), IMO 2010 (27-11-2010), je.triste (24-03-2011), kidlovecrazy (14-02-2011), ngoduchung8A (13-07-2012), perfectstrong (07-02-2011), superhuy (30-11-2010), Thanh Ngoc (18-10-2010), tinykidpro (24-02-2011), toanhoc94 (01-02-2011), vinh7aa (26-02-2014)
Old 21-08-2010, 04:51 PM   #8
Messi_ndt
+Thành Viên+
 
Tham gia ngày: Mar 2010
Bài gởi: 118
Thanks: 188
Thanked 90 Times in 39 Posts
Gửi tin nhắn qua Yahoo chát tới Messi_ndt
Trích:
Nguyên văn bởi khanh.kid View Post
Cho x,y,z>0,xyz=1
c/m
$18( \frac{1}{x^3 +1}+\frac{1}{y^3 +1}+\frac{1}{z^3 +1}) \le (x+y+z)^3 $
Thay 1 bởi abc trên tử của vế trái để BDT thuân nhất. Đặt bút phấn tích tổng các bình phương thì có ngay $S_a,S_b,S_c >0 $
[RIGHT][I][B]Nguồn: MathScope.ORG[/B][/I][/RIGHT]
 
__________________
Kiếm Tiền Đi Thi
Messi_ndt is offline  
The Following 4 Users Say Thank You to Messi_ndt For This Useful Post:
glacial (29-03-2011), hoangnamb (24-11-2010), IMO 2010 (27-11-2010), perfectstrong (07-02-2011)
Old 21-08-2010, 11:36 PM   #9
Thanh vien
+Thành Viên+
 
Thanh vien's Avatar
 
Tham gia ngày: Oct 2009
Bài gởi: 120
Thanks: 68
Thanked 70 Times in 40 Posts
Trích:
Nguyên văn bởi khanh.kid View Post
Cho x,y,z>0,xyz=1
c/m
$18( \frac{1}{x^3 +1}+\frac{1}{y^3 +1}+\frac{1}{z^3 +1}) \le (x+y+z)^3 $
Ta có Vế trái:
$\\\le9\left(\frac1{x\sqrt x}+\frac1{y\sqrt y}+\frac1{z\sqrt z}\right)\\=9\left[(\sqrt{yz})^3+(\sqrt{zx})^3+(\sqrt{xy})^3\right] $

Cần chứng minh $9\left[(\sqrt{yz})^3+(\sqrt{zx})^3+(\sqrt{xy})^3\right]\le(x+y+z)^3 $. Đặt căn cho mất căn đi thì thành:
$(a^2+b^2+c^2)^3\ge9(a^3b^3+b^3c^3+c^3a^3) $
Cái này S.O.S ra chắc đúng
[RIGHT][I][B]Nguồn: MathScope.ORG[/B][/I][/RIGHT]
 
Thanh vien is offline  
The Following 8 Users Say Thank You to Thanh vien For This Useful Post:
glacial (29-03-2011), h.linhpk (26-03-2011), ha linh (02-02-2011), hoangnamb (24-11-2010), IMO 2010 (27-11-2010), je.triste (24-03-2011), kidlovecrazy (14-02-2011), ngoduchung8A (13-07-2012)
Old 22-08-2010, 08:40 AM   #10
truytimmattroi
+Thành Viên+
 
Tham gia ngày: Aug 2010
Đến từ: mặt trăng
Bài gởi: 6
Thanks: 3
Thanked 6 Times in 5 Posts
Bất đẳng thức

Mình có bài này muốn nhờ các bạn giúp đỡ:
Tìm hằng số k lớn nhất sao cho với mọi n nguyên dương ta có:
{n$\sqrt{3} $}$\ge $$\frac{k}{n\sqrt{3}} $
[RIGHT][I][B]Nguồn: MathScope.ORG[/B][/I][/RIGHT]
 
truytimmattroi is offline  
The Following 2 Users Say Thank You to truytimmattroi For This Useful Post:
hoangnamb (24-11-2010), IMO 2010 (27-11-2010)
Old 22-08-2010, 09:07 AM   #11
NHTRANG
+Thành Viên+
 
NHTRANG's Avatar
 
Tham gia ngày: Jul 2010
Bài gởi: 17
Thanks: 12
Thanked 10 Times in 2 Posts
Ta có bđt <=> n$\sqrt{3} $ - $\frac{k}{n\sqrt{3}} $
$\ge $[n$\sqrt{3} $]
<=>3$n^2 $+$\frac{k^2}{3n^2} $ - 2k $\ge $${[n\sqrt{3}]}^2 $
Thấy với mọi n thì 3$n^2 $ và 3$n^2 $-1 đều không là số cp. Nhưng tồn tại vô số n để 3$n^2 $-2 là scp. Do đó nếu k>1 thì tồn tại n đủ lớn để 3$n^2 $+$\frac{k^2}{3n^2} $ - 2k<3$n^2 $-2 =${[n\sqrt{3}]}^2 $
Vạy k$\le $1.Dễ thấy k=1 luôn t/m =>k=1 là gtrị cần tìm
[RIGHT][I][B]Nguồn: MathScope.ORG[/B][/I][/RIGHT]
 
__________________
Hạnh phúc là được cho đi và nhận lại nụ cười

thay đổi nội dung bởi: NHTRANG, 22-08-2010 lúc 09:10 AM
NHTRANG is offline  
The Following 6 Users Say Thank You to NHTRANG For This Useful Post:
ha linh (02-02-2011), hoangnamb (24-11-2010), IMO 2010 (27-11-2010), ngoduchung8A (13-07-2012), nhox12764 (07-12-2010), truytimmattroi (22-08-2010)
Old 22-08-2010, 09:15 AM   #12
novae
+Thành Viên Danh Dự+
 
novae's Avatar
 
Tham gia ngày: Jul 2010
Đến từ: Event horizon
Bài gởi: 2,453
Thanks: 53
Thanked 3,057 Times in 1,288 Posts
Trích:
Nguyên văn bởi NHTRANG View Post
$3 n^2-2 ={[n\sqrt{3}]}^2 $
lời giải sai ở chỗ này, vd cho n=14 thì $3n^2-2=586; {[n\sqrt{3}]}^2=24^2=576 $
đáp số đúng hình như là $\sqrt3(\sqrt3-1) $
[RIGHT][I][B]Nguồn: MathScope.ORG[/B][/I][/RIGHT]
 
__________________
M.

thay đổi nội dung bởi: novae, 22-08-2010 lúc 09:18 AM
novae is offline  
The Following 2 Users Say Thank You to novae For This Useful Post:
hoangnamb (24-11-2010), IMO 2010 (27-11-2010)
Old 22-08-2010, 09:18 AM   #13
maththunder
+Thành Viên+
 
Tham gia ngày: Aug 2010
Bài gởi: 2
Thanks: 0
Thanked 3 Times in 2 Posts
Một bài tìm điểm rơi

Cho : x;y;z thực
$xy + yz + 3zx = 1 $
Tìm min:
$x^2 + y^2 + z^2 $
[RIGHT][I][B]Nguồn: MathScope.ORG[/B][/I][/RIGHT]
 

thay đổi nội dung bởi: maththunder, 22-08-2010 lúc 09:43 AM
maththunder is offline  
The Following 2 Users Say Thank You to maththunder For This Useful Post:
hoangnamb (24-11-2010), IMO 2010 (27-11-2010)
Old 22-08-2010, 09:28 AM   #14
crystal_liu
+Thành Viên+
 
crystal_liu's Avatar
 
Tham gia ngày: Aug 2010
Đến từ: Akaban
Bài gởi: 353
Thanks: 94
Thanked 199 Times in 141 Posts
Trích:
Nguyên văn bởi NHTRANG View Post
Ta có bđt <=> n$\sqrt{3} $ - $\frac{k}{n\sqrt{3}} $
$\ge $[n$\sqrt{3} $]
<=>3$n^2 $+$\frac{k^2}{3n^2} $ - 2k $\ge $${[n\sqrt{3}]}^2 $
Thấy với mọi n thì 3$n^2 $ và 3$n^2 $-1 đều không là số cp. Nhưng tồn tại vô số n để 3$n^2 $-2 là scp. Do đó nếu k>1 thì tồn tại n đủ lớn để 3$n^2 $+$\frac{k^2}{3n^2} $ - 2k<3$n^2 $-2 =${[n\sqrt{3}]}^2 $
Vạy k$\le $1.Dễ thấy k=1 luôn t/m =>k=1 là gtrị cần tìm
Bài sai chỗ lý luận cuối chỗ n đủ lớn ấy ,xem lại nhé
[RIGHT][I][B]Nguồn: MathScope.ORG[/B][/I][/RIGHT]
 
crystal_liu is offline  
The Following 4 Users Say Thank You to crystal_liu For This Useful Post:
hoangnamb (24-11-2010), IMO 2010 (27-11-2010), je.triste (24-03-2011), vu thanh tung (08-01-2011)
Old 22-08-2010, 09:38 AM   #15
NHTRANG
+Thành Viên+
 
NHTRANG's Avatar
 
Tham gia ngày: Jul 2010
Bài gởi: 17
Thanks: 12
Thanked 10 Times in 2 Posts
Trích:
Nguyên văn bởi novae View Post
lời giải sai ở chỗ này, vd cho n=14 thì $3n^2-2=586; {[n\sqrt{3}]}^2=24^2=576 $
đáp số đúng hình như là $\sqrt3(\sqrt3-1) $
Xét dãy ($x_0;y_0 $)=(1,1); ($x_{k+1};y_{k+1} $)=$2x_k +3y_k ; x_k+2y_k $
Dãy tren tăng vô hạn và mọi số hạng của dãy đều t/m3$y^2 $-2=$x^2 $ (=> x=[y$\sqrt{3} $]
Do đó có thể nói tồn tại n đủ lớn để 3$n^2 $-2 là số chính phuơng và2k>2+$\frac{k^2}{3n^2} $. Giá trị đó của n sễ không t/m bài toán.
Do vậy khi k>1(bao gồm cả$\sqrt3(\sqrt3-1) $) sẽ không được.
[RIGHT][I][B]Nguồn: MathScope.ORG[/B][/I][/RIGHT]
 
__________________
Hạnh phúc là được cho đi và nhận lại nụ cười

thay đổi nội dung bởi: NHTRANG, 22-08-2010 lúc 09:57 AM
NHTRANG is offline  
The Following 4 Users Say Thank You to NHTRANG For This Useful Post:
hoangnamb (24-11-2010), IMO 2010 (27-11-2010), ngoduchung8A (13-07-2012), truytimmattroi (23-08-2010)
Ðề tài đã khoá Gởi Ðề Tài Mới

Bookmarks

Tags
bất đẳng thức

Ðiều Chỉnh
Xếp Bài

Quuyền Hạn Của Bạn
You may not post new threads
You may not post replies
You may not post attachments
You may not edit your posts

BB code is Mở
Smilies đang Mở
[IMG] đang Mở
HTML đang Tắt

Chuyển đến


Múi giờ GMT. Hiện tại là 07:17 PM.


Powered by: vBulletin Copyright ©2000-2024, Jelsoft Enterprises Ltd.
Inactive Reminders By mathscope.org
[page compression: 107.90 k/124.68 k (13.46%)]